Đến nội dung

Hình ảnh

Topic: [LTDH] Mỗi ngày hai bất đẳng thức.


  • Chủ đề bị khóa Chủ đề bị khóa
Chủ đề này có 215 trả lời

#161
tritanngo99

tritanngo99

    Đại úy

  • Điều hành viên THPT
  • 1644 Bài viết

Bài 72: anhquannbk đã cho lời giải đúng. Bài 71: Có rất nhiều lời giải đúng từ hanguyen45, Namvip, anhquannbk.

Dưới đây là cách khác bài 71:

Ta có: $a^3+b^3+c^3=(a+b+c)^3-3(a+b)(b+c)(c+a)\le 27-3.8abc=27-24abc$.

Như vậy, suy ra: $\sqrt[3]{\frac{a^3+b^3+c^3}{3}}+8\sqrt[3]{abc}\le \sqrt[3]{\frac{27-24abc}{3}}+8\sqrt[3]{abc}=\sqrt[3]{9-8abc}+8\sqrt[3]{abc}$.

Đặt $t=abc \implies t\in (0;1]$.

Xét hàm $f(t)=\sqrt[3]{9-8t}+8\sqrt[3]{t}\text{  }\forall t\in (0;1]$.

Ta có: $f'(t)=\frac{8}{3}[\frac{1}{\sqrt[3]{t^2}}-\frac{1}{\sqrt[3]{(9-8t)^2}}]\ge 0\forall t\in (0;1]$.

$\implies f(t)\le f(1)=9\implies Q.E.D$.



#162
tritanngo99

tritanngo99

    Đại úy

  • Điều hành viên THPT
  • 1644 Bài viết

Tiếp theo: 

Bài 73: Cho $a,b,c>0$. Chứng minh rằng: $\sum_{cyc}\frac{a^2+b^2}{a+b}\ge \sqrt{3(a^2+b^2+c^2)}$.

Bài 74: Cho $x,y,z\ge 0$ sao cho không có $2$ số nào cùng bằng không. Chứng minh rằng:

$\frac{x^2+y^2+z^2}{xy+yz+zx}+\frac{\sqrt{xy}}{x+y}+\frac{\sqrt{yz}}{y+z}\ge 2$

 



#163
baopbc

baopbc

    Himura Kenshin

  • Thành viên nổi bật 2016
  • 410 Bài viết

Bài 73. Ta có : $\frac{a^2+b^2}{a+b}-\frac{a+b}{2}=\frac{2a^2+2b^2-(a+b)^2}{2(a+b)}=\frac{(a-b)^2}{2(a+b)}$

Tương tự với các biểu thức còn lại ta suy ra :

\[\sum \frac{a^2+b^2}{a+b}-(a+b+c)=\sum \frac{(a-b)^2}{2(a+b)}\]

Mặt khác theo bất đẳng thức $\text{Cauchy}$ thì :

\[\sqrt{3(a^2+b^2+c^2)}-(a+b+c)=\frac{\sum (a-b)^2}{\sqrt{3(a^2+b^2+c^2)}+(a+b+c)}\leq \frac{\sum (a-b)^2}{2(a+b+c)}\]

Do đó ta cần chứng minh :

\[\sum (a-b)^2(\frac{1}{2(a+b)}-\frac{1}{2(a+b+c)})\geq 0\]

Do $a,b,c>0$ nên bất đẳng thức này hiển nhiên đúng.

Dấu đẳng thức xảy ra khi và chỉ khi $a=b=c$.

 



#164
Namvip

Namvip

    Hạ sĩ

  • Thành viên
  • 50 Bài viết

Bài 73 : Lời giải khác 

Ta có 

$\sum \frac{a^{2}+b^{2}}{a+b}\geq \frac{\left ( \sum \sqrt{a^{2}+b^{2}} \right )^{2}}{2(a+b+c)}$

Lại có 

$(\sum \sqrt{a^{2}+b^{2}})^{2}=2(a^{2}+b^{2}+c^{2})+\sum 2\sqrt{(a^{2}+b^{2})(a^{2}+c^{2})}\geq 2(a^{2}+b^{2}+c^{2})+2\sum (a^{2}+bc)=3(a^{2}+b^{2}+c^{2})+(a+b+c)^{2}$

mà 

$3(a^{2}+b^{2}+c^{2})+(a+b+c)^{2}\geq 2(a+b+c)\sqrt{3(a^{2}+b^{2}+c^{2})}$

=>$\sum \frac{a^{2}+b^{2}}{a+b}\geq \frac{2(a+b+c)\sqrt{3(a^{2}+b^{2}+c^{2})}}{2(a+b+c)}=\sqrt{3(a^{2}+b^{2}+c^{2})}$



#165
vpvn

vpvn

    Binh nhất

  • Thành viên mới
  • 41 Bài viết

BÀI 74 để ý $\frac{x^{2}+y^{2}+z^{2}}{xy+yz+zx}\geq \frac{x^{2}+y^{2}+z^{2}+y^{2}}{xy+yz+zx+y^{2}}$

                     =$\frac{x^{2}+2y^{2}+z^{2}}{(x+y)(y+z)}$

suy ra  VT =$\frac{x^{2}+2y^{2}+z^{2}+(x+y)\sqrt{xy}+(y+z)\sqrt{yz}}{(x+y)(y+z)}$ 

                  $\geq$ $\frac{x^{2}+2y^{2}+z^{2}+2xy+2yz}{(x+y)(y+z)}$

                 =$\frac{(x+y)^{2}+(y+z)^{2}}{(x+y)(y+z)}$

                   $\geq$ 2

Dấu bằng xảy ra $\Leftrightarrow$ x=z , y=0



#166
Lequynhdiep

Lequynhdiep

    Binh nhì

  • Thành viên mới
  • 13 Bài viết

Bài 73. Ta có : $\frac{a^2+b^2}{a+b}-\frac{a+b}{2}=\frac{2a^2+2b^2-(a+b)^2}{2(a+b)}=\frac{(a-b)^2}{2(a+b)}$

Tương tự với các biểu thức còn lại ta suy ra :

\[\sum \frac{a^2+b^2}{a+b}-(a+b+c)=\sum \frac{(a-b)^2}{2(a+b)}\]

Mặt khác theo bất đẳng thức $\text{Cauchy}$ thì :

\[\sqrt{3(a^2+b^2+c^2)}-(a+b+c)=\frac{\sum (a-b)^2}{\sqrt{3(a^2+b^2+c^2)}+(a+b+c)}\leq \frac{\sum (a-b)^2}{2(a+b+c)}\]

Do đó ta cần chứng minh :

\[\sum (a-b)^2(\frac{1}{2(a+b)}-\frac{1}{2(a+b+c)})\geq 0\]

Do $a,b,c>0$ nên bất đẳng thức này hiển nhiên đúng.

Dấu đẳng thức xảy ra khi và chỉ khi $a=b=c$.

tại sao lại nghĩ đến - (a+b/2) vậy ạ?



#167
tritanngo99

tritanngo99

    Đại úy

  • Điều hành viên THPT
  • 1644 Bài viết

Bài 73bài 74: baopbc, Namvip,vpvn đã cho lời giải đúng. 

Mình xin trình bày cách khác bài 74( Dài hơn bạn vpvn).

Đầu tiên đi CM: $\frac{\sqrt{xy}}{x+y}\ge \frac{2xy}{(x+y)^2}(*)$.

Thật vậy: $(*)\iff \sqrt{xy}(x+y-2\sqrt{xy})\ge 0(TRUE)$.

Dấu $=$ xảy ra khi $x=0\text{ hoặc }y=0\text{ hoặc }x=y$.

Tương tự: $\frac{\sqrt{yz}}{y+z}\ge \frac{2yz}{(y+z)^2}$

$\implies \frac{\sqrt{xy}}{x+y}+\frac{\sqrt{yz}}{y+z}\ge \frac{2xy}{(x+y)^2}+\frac{2yz}{(y+z)^2}$.

Vậy ta đi CM: $\frac{x^2+y^2+z^2}{xy+yz+zx}+\frac{2xy}{(x+y)^2}+\frac{2yz}{(y+z)^2}\ge 2$.

$\iff [\frac{x^2+y^2+z^2}{xy+yz+zx}+\frac{4xy}{(x+y)^2}]+ [\frac{x^2+y^2+z^2}{xy+yz+zx}+\frac{4yz}{(y+z)^2}]\ge 4$.

Ta sẽ CM: $\frac{x^2+y^2+z^2}{xy+yz+zx}+\frac{4xy}{(x+y)^2}\ge 2\iff \frac{(x+y+z)^2}{xy+yz+zx}+\frac{4xy}{(x+y)^2}\ge 4$.

$\iff \frac{(x+y+z)^2}{xy+yz+zx}\ge 4[1-\frac{xy}{(x+y)^2}]\iff \frac{(x+y+z)^2}{xy+yz+zx}\ge \frac{4(x^2+xy+y^2)}{(x+y)^2}$.

$\iff (x+y+z)^2(x+y)^2\ge 4(xy+yz+zx)(x^2+xy+y^2)\iff (xy+yz+zx+x^2+xy+y^2)^2\ge 4(xy+yz+zx)(x^2+xy+y^2)(TRUE)$.

Tương tự ta có: DPCM



#168
tritanngo99

tritanngo99

    Đại úy

  • Điều hành viên THPT
  • 1644 Bài viết

Tiếp theo:

Bài 75: Cho $a,b,c$ là độ dài $3$ cạnh tam giác thỏa mãn: $a+b+c=3$.

Chứng minh rằng: $\frac{1}{a}+\frac{1}{b}+\frac{1}{c}+\frac{19}{12}\ge \frac{6(a^4+b^4+c^4)}{7}$

Bài 76: Cho $a,b,c>0$. Chứng minh rằng:

$(\frac{a^2}{b^2+c^2}+\frac{1}{3})(\frac{b^2}{c^2+a^2}+\frac{1}{3})(\frac{c^2}{a^2+b^2}+\frac{1}{3})\ge (\frac{a}{b+c}+\frac{1}{3})(\frac{b}{c+a}+\frac{1}{3})(\frac{c}{a+b}+\frac{1}{3})$


Bài viết đã được chỉnh sửa nội dung bởi tritanngo99: 04-10-2016 - 22:04


#169
anhquannbk

anhquannbk

    Sĩ quan

  • Thành viên
  • 477 Bài viết

Bài 76:

Đặt $ x=\dfrac{a^2}{b^2+c^2}, y= \dfrac{b^2}{a^2+c^2}, z=\dfrac{c^2}{a^2+b^2}$

$ m=\dfrac{a}{b+c}, n= \dfrac{b}{a+c}, p=\dfrac{c}{a+b}$

Bất đẳng thức cần chứng minh: $ (x+\dfrac{1}{3})(y+\dfrac{1}{3})(z+\dfrac{1}{3}) \ge (m+\dfrac{1}{3})(n+\dfrac{1}{3})(p+\dfrac{1}{3}) $

$ \iff xyz +\dfrac{1}{3}(xy+yz+xz)+\dfrac{1}{9}(x+y+z) \ge mnp + \dfrac{1}{3}(mn+np+pm)+\dfrac{1}{9}(m+n+p)$

Ta một bất đẳng thức quen thuộc sau: $ \dfrac{a^2}{b^2+c^2} + \dfrac{b^2}{a^2+c^2} + \dfrac{c^2}{a^2+b^2} \ge \dfrac{a}{b+c} +\dfrac{b}{a+c} +\dfrac{c}{a+b} $, tức $ x+y+z \ge m+n+p $

Ta chứng minh $ xyz \ge mnp $, tức $ \dfrac{a^2b^2c^2}{(a^2+b^2)(b^2+c^2)(c^2+a^2)} \ge \dfrac{abc}{(a+b)(b+c)(c+a)} $

$ \iff (a^2+b^2)(b^2+c^2)(c^2+a^2) \ge abc(a+b)(b+c)(c+a) $

$ \iff  c^2(a-b)^2(a^2+ab+b^2) + b^2(a-c)^2(a^2+ac+c^2)+ a^2(b-c)^2(b^2+bc+c^2) \ge 0 $ (đúng)

Tương tự ta chứng minh được $ xy+yz+xz \ge mn+mp+np $

suy ra đpcm.



#170
tritanngo99

tritanngo99

    Đại úy

  • Điều hành viên THPT
  • 1644 Bài viết

Bài 76: anhquannbk đã cho lời giải đúng. Dưới đây là lời giải bài 75:

Với $a,b,c$ là ba cạnh tam giác ta có bất đẳng thức quen thuộc sau:

 $2(a^2b^2+b^2c^2+c^2a^2)-(a^4+b^4+c^4)=(a+b+c)(a+b-c)(b+c-a)(c+a-b)\ge 0$

$\implies (a^2+b^2+c^2)^2\ge 2(a^4+b^4+c^4)$.

Ta cần chứng minh: $\frac{1}{a}+\frac{1}{b}+\frac{1}{c}+\frac{19}{12}\ge \frac{3(a^2+b^2+c^2)^2}{7}$.

Đặt $(a;b;c)\to (y+z;z+x;x+y)\implies x,y,z>0;x+y+z=\frac{3}{2}$.

Bất đẳng thức trở thành: $\frac{1}{y+z}+\frac{1}{z+x}+\frac{1}{x+y}+\frac{19}{12}\ge \frac{12(x^2+y^2+z^2+xy+yz+zx)^2}{7}$.

Đặt $4(xy+yz+zx)=t\implies 3\ge t>0$.

Ta có: $x^2+y^2+z^2+xy+zx+xy=(x+y+z)^2-(xy+yz+zx)=\frac{9-t}{4}$.

Nhân cả $2$ vế với $x+yz$: $(x+y+z)(\frac{1}{y+z}+\frac{1}{z+x}+\frac{1}{x+y})=3+\frac{x}{y+z}+\frac{y}{z+x}+\frac{z}{x+y}$

$\ge 3+\frac{x^2+y^2+z^2}{xy+yz+zx}=1+\frac{9}{t}$.

Ta chỉ cần CM: $\frac{9}{t}+\frac{27}{8}\ge \frac{9(9-t)^2}{56}\iff (t-2)^2(t-14)\le 0$.

Bất đẳng thức này hiển nhiên đúng.



#171
tritanngo99

tritanngo99

    Đại úy

  • Điều hành viên THPT
  • 1644 Bài viết

Tiếp theo: 

Bài 77: Cho $a,b,c>0$ thỏa mãn: $a+b+c=3$. Chứng minh rằng:

$\frac{1}{1+ab}+\frac{1}{1+bc}+\frac{1}{1+ca}\ge \frac{9}{2(\sqrt{a}+\sqrt{b}+\sqrt{c})}$.

Bài 78: Cho $x,y,z$ là các số thực dương thỏa mãn: $x\le z$. Tìm GTNN của biểu thức:

$P=\sqrt{2+\frac{2x^2}{(x+y)^2}-\frac{2z(2y+z)}{(y+z)^2}}+\frac{3z}{z+x}$.



#172
tritanngo99

tritanngo99

    Đại úy

  • Điều hành viên THPT
  • 1644 Bài viết

Dưới đây là lời giải bài 77 và bài 78:

Lời giải bài 77:

Ta có: $\frac{1}{1+ab}=1-\frac{ab}{ab+1}\ge 1-\frac{\sqrt{ab}}{2}$.

$\implies \sum \frac{1}{1+ab}\ge 3-\frac{\sqrt{ab}+\sqrt{bc}+\sqrt{ca}}{2}=3-\frac{(\sqrt{a}+\sqrt{b}+\sqrt{c})^2-a-b-c}{4}.$

Ta cân chứng minh: $\frac{15}{4}-\frac{(\sum \sqrt{a})^2}{4}\ge \frac{9}{2\sqrt{a}}$.

Đặt $t=\sqrt{a}+\sqrt{b}+\sqrt{c}$. Để ý rằng: $\sqrt{a+b+c}\le \sqrt{a}+\sqrt{b}+\sqrt{c}\le \sqrt{3(a+b+c)}$.

Nên: $\sqrt{3}<t\le 3$. Khi đó BDT cần chứng minh viết lại thành:

$\frac{15}{4}-\frac{t^2}{4}\ge \frac{9}{2t}\iff (t-3)(t-\frac{\sqrt{33}-\sqrt{3}}{2})(t+\frac{\sqrt{33}-\sqrt{3}}{2})\le 0$.

Dễ thấy: $\sqrt{3}>\frac{\sqrt{33}-\sqrt{3}}{2}\implies Q.E.D$.

Lời giải bài 78:

Viết P lại dưới dạng: $P=\sqrt{2(\frac{x^2}{(x+y)^2})+\frac{y^2}{(y+z)^2}}+\frac{3z}{z+x}\ge \frac{x}{x+y}+\frac{y}{y+z}+\frac{3z}{z+x}$.

Đặt: $(\frac{x}{y};\frac{z}{y})\to (a;b)(a\le b)$.

Như vậy thì ta có: $P\ge \frac{a}{a+1}+\frac{1}{1+b}+\frac{3b}{a+b}=f(a,b)$.

Đến đây có nhiều cách giải: Có thể đạo hàm hoặc chứng minh trực tiếp:

$f(a,b)-\frac{5}{2}=\frac{(b-a)(3ab+a+b+3)}{2(a+b)(a+1)(b+1)}\ge 0$.

Vậy $P_{min}=\frac{5}{2}$. Đẳng thức xảy ra tại: $x=z$.



#173
tritanngo99

tritanngo99

    Đại úy

  • Điều hành viên THPT
  • 1644 Bài viết

Tiếp theo:

Bài 79: Cho các số dương $x,y,z$ thỏa mãn: $x+y+z=3$. Chứng minh rằng:

$P=\sum \frac{4x}{y(2\sqrt{1+8y^3}+4x-2)}\ge \frac{3}{2}$.

Bài 80: Cho $a \ge b\ge c>0$. Chứng minh rằng: $P=\frac{(3ab+bc)^2}{b^4}+\frac{121b^2}{a^2+b^2+c^2+8ac}\ge 27$.


Bài viết đã được chỉnh sửa nội dung bởi tritanngo99: 06-10-2016 - 17:49


#174
vpvn

vpvn

    Binh nhất

  • Thành viên mới
  • 41 Bài viết

nhận xét $2\sqrt{1+8y^{3}}\leq 2+4y^{2}$ (am gm)

$\Rightarrow$ P $\geq \sum \frac{x}{y(y^{2}+x)}$ =$\sum \frac{1}{y}-\sum \frac{y}{x+y^{2}}$

$\geq \sum \frac{1}{x}-\frac{1}{2}\sum \frac{1}{\sqrt{x}}$ (do x+y2 $\geq$ 2y$\sqrt{x}$) 

mà $\sum \frac{1}{\sqrt{x}}\leq \sqrt{3(\sum \frac{1}{x})}$

$\Rightarrow$ P $\geq \sum \frac{1}{x}-\frac{1}{2}\sqrt{3(\sum \frac{1}{x})}$ và $\sum \frac{1}{x}\geq 3$ $\Rightarrow$ P $\geq$ $\frac{3}{2}$



#175
PlanBbyFESN

PlanBbyFESN

    Thiếu úy

  • Điều hành viên OLYMPIC
  • 637 Bài viết

Bài 80: Cho $a \ge b\ge c>0$. Chứng minh rằng: $P=\frac{(3ab+bc)^2}{b^4}+\frac{121b^2}{a^2+b^2+c^2+8ac}\ge 27$.

 

$\frac{(3ab+bc)^2}{b^4}+\frac{121b^2}{a^2+b^2+c^2+8ac}\geq 27\Leftrightarrow \frac{(3a+c)^2}{b^2}+\frac{121b^2}{a^2+b^2+c^2+8ac}\geq 27$

 

$\Leftrightarrow  \frac{9a^{2}+6ac+c^{2}}{b^2}+\frac{121b^2}{a^2+b^2+c^2+8ac}\geq 27$

 

Mà vì $a\geq b\geq c>0 \Rightarrow 9a^{2}\geq a^{2}+b^{2}+2ac+5b^{2}$

 

$\Rightarrow \frac{9a^{2}+6ac+c^{2}}{b^2}+\frac{121b^2}{a^2+b^2+c^2+8ac}\geq \frac{a^{2}+b^{2}+c^{2}+8ac+5b^{2}}{b^2}+\frac{121b^2}{a^2+b^2+c^2+8ac}\geq 5+\left (\frac{a^{2}+b^{2}+c^{2}+8ac}{b^2}+\frac{121b^2}{a^2+b^2+c^2+8ac} \right )\geq 27$    (Am-Gm)


Bài viết đã được chỉnh sửa nội dung bởi PlanBbyFESN: 06-10-2016 - 21:41

:huh:


#176
PlanBbyFESN

PlanBbyFESN

    Thiếu úy

  • Điều hành viên OLYMPIC
  • 637 Bài viết

Bài 79: Cho các số dương $x,y,z$ thỏa mãn: $x+y+z=3$. Chứng minh rằng:

$P=\sum \frac{4x}{y(2\sqrt{1+8y^3}+4x-2)}\ge \frac{3}{2}$.

 

Nhìn qua chắc hướng bạn trên đúng rồi, mình trình bày rõ ra cho mọi người dễ đọc, bạn trình bày hơi khó nhìn

 

$.\sqrt{1+8y^3}=\sqrt{(1+2y)(1-2y+4y^{2})}\leq \sqrt{\frac{(1+2y+1-2y+4y^{2})}{4}}=2y^{2}+1$

 

$ \Rightarrow 0<y(2\sqrt{1+8y^3}+4x-2)\leq y(2(2y^{2}+1)+4x-2)=4y(y^{2}+x)$

 

$\Rightarrow \frac{4x}{y(2\sqrt{1+8y^3}+4x-2)}\geq \frac{x}{y(y^{2}+x)}\Rightarrow \sum \frac{4x}{y(2\sqrt{1+8y^3}+4x-2)}\geq \sum \frac{x}{y(y^{2}+x)}$

 

$\frac{x}{y(y^{2}+x)}= \frac{x+y^{2}-y^{2}}{y(y^{2}+x)}=\frac{1}{y}-\frac{y}{y^{2}+x}\geq \frac{1}{y}-\frac{y}{2y\sqrt{x}}\geq \frac{1}{y}-\frac{1}{8}(\frac{1}{x}+1)$                                                                                                                                                                               (Am-Gm)

 

$\Rightarrow  \sum \frac{x}{y(y^{2}+x)} \geq \sum  \left [\frac{1}{y}-\frac{1}{8}(\frac{1}{x}+1)  \right ]\geq \frac{9}{4} $     (Do $x+y+z=3$) (C-S)

 

...................


:huh:


#177
tritanngo99

tritanngo99

    Đại úy

  • Điều hành viên THPT
  • 1644 Bài viết

Hai bài 79 và 80: vpvnPlanBbyFESN đã cho lời giải đúng. Mình xin tiếp tục:

Bài 81: Cho các số thực thỏa mãn: $x,y,z>0$ và thỏa mãn: $x=y+z+xyz$. Tìm GTLN của biểu thức:

$P=\frac{(z+z\sqrt{xy})^2}{(x+y)(z^2+1)}+\frac{2z}{(z^2+1)\sqrt{z^2+1}}$.

Bài 82: Cho $a,b$ là các số dương và thỏa mãn: $a^2+b^2=a+b$. Tìm GTNN của biểu thức:

$P=3a+2b+\frac{16}{\sqrt{a+3b}}+\frac{16}{\sqrt{3a+1}}$



#178
Nam Antoneus

Nam Antoneus

    Hạ sĩ

  • Thành viên
  • 58 Bài viết

Bài 82: Cho $a,b$ là các số dương và thỏa mãn: $a^2+b^2=a+b$. Tìm GTNN của biểu thức:

$P=3a+2b+\frac{16}{\sqrt{a+3b}}+\frac{16}{\sqrt{3a+1}}$

$P=(a+3b+\frac{8}{\sqrt{a+3b}}+\frac{8}{\sqrt{a+3b}})+(3a+1+\frac{8}{\sqrt{3a+1}}+\frac{8}{\sqrt{3a+1}})-a-b-1$

$P\geqslant 3\sqrt[3]{(a+3b).\frac{8}{\sqrt{a+3b}}.\frac{8}{\sqrt{a+3b}}}+3\sqrt[3]{(3a+1).\frac{8}{\sqrt{3a+1}}.\frac{8}{\sqrt{3a+1}}}-a-b-1$

$P\geqslant 3.4+3.4-a-b-1=23-(a+b)$

Lại có: $(a+b)^2\leqslant 2(a^2+b^2)=2(a+b)\Leftrightarrow a+b\leq 2$

Vậy $P\geq 21$



#179
NTA1907

NTA1907

    Thượng úy

  • Thành viên
  • 1014 Bài viết

Hai bài 79 và 80: vpvnPlanBbyFESN đã cho lời giải đúng. Mình xin tiếp tục:

Bài 81: Cho các số thực thỏa mãn: $x,y,z>0$ và thỏa mãn: $x=y+z+xyz$. Tìm GTLN của biểu thức:

$P=\frac{(z+z\sqrt{xy})^2}{(x+y)(z^2+1)}+\frac{2z}{(z^2+1)\sqrt{z^2+1}}$.

Áp dụng bất đẳng thức Cauchy-Schwarz ta có:

$P=\frac{z(z+xyz+2z\sqrt{xy})}{(x+y)(z^{2}+1)}+\frac{2z}{(z^{2}+1)\sqrt{z^{2}+1}}=\frac{z(x-y+2z\sqrt{xy})}{(x+y)(z^{2}+1)}+\frac{2z}{(z^{2}+1)\sqrt{z^{2}+1}}=\frac{z\left [ (x-y).1+2\sqrt{xy}.z \right ]}{(x+y)(z^{2}+1)}+\frac{2z}{(z^{2}+1)\sqrt{z^{2}+1}}\leq \frac{z\sqrt{\left [ (x-y)^{2}+4xy \right ](1+z^{2})}}{(x+y)(1+z^{2})}+\frac{2z}{(z^{2}+1)\sqrt{z^{2}+1}}=\frac{z}{\sqrt{z^{2}+1}}+\frac{2z}{(z^{2}+1)\sqrt{z^{2}+1}}=\frac{z}{\sqrt{z^{2}+1}}+\frac{2z}{\sqrt{z^{2}+1}}\left ( 1-\frac{z^{2}}{z^{2}+1} \right )=\frac{3z}{\sqrt{z^{2}+1}}-\frac{2z^{3}}{(z^{2}+1)\sqrt{z^{2}+1}}$

Khảo sát hàm số $f(t)=3t-2t^{3}$ với $0\leq t=\frac{z}{\sqrt{z^{2}+1}}\leq \frac{1}{\sqrt{2}}$

$\Rightarrow P\leq f(t)\leq \sqrt{2}$

Dấu "=" xảy ra$\Leftrightarrow x=\sqrt{2}+1, y=\sqrt{2}-1, z=1$


Vũ trụ không có biên trong không gian, không có bắt đầu và kết thúc trong thời gian và chẳng có việc gì cho đấng sáng thế phải làm ở đây cả.

 


#180
tritanngo99

tritanngo99

    Đại úy

  • Điều hành viên THPT
  • 1644 Bài viết

Bài 81 và 82: Nam AntonuesNTA 1907 đã cho lời giải đúng. MÌnh xin tiếp tục:

Bài 83: Cho $x,y,z$ là các số thực dương thỏa mãn: $x^2+y^2+6z^2=4z(x+y)$. Tìm GTNN của biểu thức:

$P=\frac{x^3}{y(x+z)^2}+\frac{y^3}{x(y+z)^2}+\frac{\sqrt{x^2+y^2}}{z}$.

Bài 84: Cho $a,b,c$ là các số thực dương thỏa mãn: $ab\ge \frac{7}{3}$ và $3a+57b+7c=3abc+\frac{100}{a}$. Tìm GTNN của: $P=a+b+c$.






1 người đang xem chủ đề

0 thành viên, 1 khách, 0 thành viên ẩn danh